K
Khách

Hãy nhập câu hỏi của bạn vào đây, nếu là tài khoản VIP, bạn sẽ được ưu tiên trả lời.

6 tháng 11 2017

Nội qui tham gia "Giúp tôi giải toán"

1. Không đưa câu hỏi linh tinh lên diễn đàn, chỉ đưa các bài mà mình không giải được hoặc các câu hỏi hay lên diễn đàn;

2. Không trả lời linh tinh, không phù hợp với nội dung câu hỏi trên diễn đàn.

3. Không "Đúng" vào các câu trả lời linh tinh nhằm gian lận điểm hỏi đáp.

Các bạn vi phạm 3 điều trên sẽ bị giáo viên của Online Math trừ hết điểm hỏi đáp, có thể bị khóa tài khoản hoặc bị cấm vĩnh viễn không đăng nhập vào trang web.

mong các bn đừng làm như vậy nah

2 tháng 5 2016

ta có \(\frac{1}{x^2+x}+\frac{x^2+x}{4}>=2\cdot\sqrt{\frac{1\cdot\left(x^2+x\right)}{\left(x^2+x\right)\cdot4}}=1\)

tương tự => \(\frac{1}{y^2+y}+\frac{y^2+y}{4}>=1;\frac{1}{z^2+z}+\frac{z^2+z}{4}>=1\)

=> VT >= 3-(\(\frac{x^2+x}{4}+\frac{y^2+y}{4}+\frac{z^2+z}{4}\))=3-\(\frac{x^2+y^2+z^2+3}{4}\)

mà \(\frac{x^2}{4}+\frac{y^2}{4}+\frac{z^2}{4}>=\frac{\left(x+y+z\right)^2}{4+4+4}=\frac{3}{4}\)

=> P>= 3-3/4-3/4=3/2

Dấu bằng khi x=y=z=1

3 tháng 5 2016

Bài bạn Lương Ngọc Anh bị ngược dấu nên sai hoàn toàn. Lời giải:

Ta có:

\(\frac{1}{x^2+x}=\frac{1}{x\left(x+1\right)}=\frac{1}{x}-\frac{1}{x+1}\)

Tương tự, ta được:

\(VT=\left(\frac{1}{x}+\frac{1}{y}+\frac{1}{z}\right)-\left(\frac{1}{x+1}+\frac{1}{y+1}+\frac{1}{z+1}\right)\)

Áp dụng BĐT Schwarz:

\(\frac{1}{x+1}+\frac{1}{y+1}+\frac{1}{z+1}\le\frac{1}{4}\left(3+\frac{1}{x}+\frac{1}{y}+\frac{1}{z}\right)=\frac{3}{4}+\frac{1}{4}\left(\frac{1}{x}+\frac{1}{y}+\frac{1}{z}\right)\)

Do đó:

\(VT\ge\frac{3}{4}\left(\frac{1}{x}+\frac{1}{y}+\frac{1}{z}\right)-\frac{3}{4}\left(1\right)\)

Mặt khác:

\(\frac{1}{x}+\frac{1}{y}+\frac{1}{z}\ge\frac{9}{x+y+z}=3\left(2\right)\)

TỪ (1) VÀ (2) TA CÓ ĐIỀU PHẢI CHỨNG MINH.

7 tháng 6 2021

\(\dfrac{x-y}{z^2+1}=\dfrac{x-y}{z^2+xy+yz+zx}=\dfrac{x-y}{z\left(z+y\right)+x\left(z+y\right)}=\dfrac{x-y}{\left(x+z\right)\left(z+y\right)}\)

Tương tự: \(\dfrac{y-z}{x^2+1}=\dfrac{y-z}{\left(x+y\right)\left(x+z\right)}\);\(\dfrac{z-x}{y^2+1}=\dfrac{z-x}{\left(x+y\right)\left(y+z\right)}\)

Cộng vế với vế \(\Rightarrow VT=\dfrac{x-y}{\left(x+z\right)\left(y+z\right)}+\dfrac{y-z}{\left(x+y\right)\left(x+z\right)}+\dfrac{z-x}{\left(x+y\right)\left(y+z\right)}\)

\(=\dfrac{\left(x-y\right)\left(x+y\right)+\left(y-z\right)\left(y+z\right)+\left(z-x\right)\left(z+x\right)}{\left(x+y\right)\left(y+z\right)\left(z+x\right)}\)

\(=\dfrac{x^2-y^2+y^2-z^2+z^2-x^2}{\left(x+y\right)\left(y+z\right)\left(z+x\right)}=0\)(đpcm)

5 tháng 11 2016

Ta chứng minh \(x^4+y^4\ge x^3y+xy^3\)

\(\Leftrightarrow x^3\left(x-y\right)-y^3\left(x-y\right)\ge0\)

\(\Leftrightarrow\left(x-y\right)^2\left(x^2+xy+y^2\right)\ge0\)

\(\Leftrightarrow\left(x-y\right)^2\left[\left(x+\frac{y}{2}\right)^2+\frac{3y^2}{4}\right]\ge0\)(luôn đúng)

Áp dụng vào bài toán ta có:

\(x^4+y^4\ge x^3y+xy^3\)\(\Rightarrow2\left(x^4+y^4\right)\ge x^4+y^4+x^3y+xy^3\)\(=\left(x^3+y^3\right)\left(x+y\right)\)

\(\Rightarrow\frac{x^4+y^4}{x^3+y^3}\ge\frac{x+y}{2}\).Tương tự ta cũng có:

\(\frac{y^4+z^4}{y^3+z^3}\ge\frac{y+z}{2};\frac{z^4+x^4}{z^3+x^3}\ge\frac{z+x}{2}\)

Cộng theo vế ta có: \(VT\ge\frac{x+y}{2}+\frac{y+z}{2}+\frac{z+x}{2}=x+y+z=1\)

Dấu = khi \(x=y=z=\frac{2008}{3}\)

27 tháng 2 2020

Bài này áp dụng BĐT này nhé , với x,y > 0 ta có :

\(\frac{1}{x}+\frac{1}{y}\ge\frac{4}{x+y}\) ( Cách chứng minh thì chuyển vế quy đồng nhé )

Áp dụng vào bài toán ta có :

\(\frac{1}{2x+y+z}=\frac{1}{4}\left(\frac{4}{\left(x+y\right)+\left(z+x\right)}\right)\le\frac{1}{4}\left(\frac{1}{x+y}+\frac{1}{z+x}\right)=\frac{1}{16}\left(\frac{4}{x+y}+\frac{4}{z+x}\right)\)

                                                           \(\le\frac{1}{16}\left(\frac{1}{x}+\frac{1}{y}+\frac{1}{z}+\frac{1}{x}\right)\)

\(\Rightarrow\frac{1}{2x+y+z}\le\frac{1}{16}\left(\frac{1}{x}+\frac{1}{y}+\frac{1}{z}+\frac{1}{x}\right)\)

Tương tự ta có :

\(\frac{1}{x+2y+z}\le\frac{1}{16}\left(\frac{1}{x}+\frac{1}{y}+\frac{1}{y}+\frac{1}{z}\right)\)

\(\frac{1}{x+y+2z}\le\frac{1}{16}\left(\frac{1}{x}+\frac{1}{y}+\frac{1}{z}+\frac{1}{z}\right)\)

Do đó : \(\frac{1}{2x+y+z}+\frac{1}{x+2y+z}+\frac{1}{x+y+2z}\le\frac{1}{16}\left(\frac{4}{x}+\frac{4}{y}+\frac{4}{z}\right)=\frac{1}{4}\left(x+y+z\right)=1\)

Dấu "=" xảy ra \(\Leftrightarrow x=y=z=\frac{3}{4}\) (đpcm)

27 tháng 2 2020

Ta có: \(\frac{1}{2x+y+z}\le\frac{1}{4}\left(\frac{1}{x+y}+\frac{1}{x+z}\right)\le\frac{1}{16}\left(\frac{2}{x}+\frac{1}{y}+\frac{1}{z}\right)\)

Tương tự: \(\frac{1}{x+2y+z}\le\frac{1}{16}\left(\frac{1}{x}+\frac{2}{y}+\frac{1}{z}\right)\)

                  \(\frac{1}{x+y+2z}\le\frac{1}{16}\left(\frac{1}{x}+\frac{1}{y}+\frac{2}{z}\right)\)

Cộng vế theo vế có: \(VT\le\frac{1}{16}\left(\frac{4}{x}+\frac{4}{y}+\frac{4}{z}\right)=1\)

14 tháng 5 2018

Ta dễ dàng chứng minh BĐT

\(x^4+y^4\ge x^3y+xy^3\)

\(\Rightarrow2\left(x^4+y^4\right)\ge x^4+y^4+x^3y+xy^3=\left(x+y\right)\left(x^3+y^3\right)\)

\(\Rightarrow\frac{x^4+y^4}{x^3+y^3}\ge\frac{x+y}{2}\)

Chứng minh tương tự, cộng theo vế, ta có:

\(\frac{x^4+y^4}{x^3+y^3}+\frac{y^4+z^4}{y^3+z^3}+\frac{z^4+x^4}{z^3+x^3}\ge\frac{x+y}{2}+\frac{y+z}{2}+\frac{z+x}{2}=\frac{2\left(x+y+z\right)}{2}=2\)

Dấu "=" xảy ra khi x=y=z=1/3

19 tháng 9 2019

Áp dụng BĐT Cauchy cho 3 số dương, ta được:

\(\frac{1}{x\left(x+1\right)}+\frac{x}{2}+\frac{x+1}{4}\ge\sqrt[3]{\frac{1}{x\left(x+1\right)}.\frac{x}{2}.\frac{x+1}{4}}=3.\sqrt{\frac{1}{4}}=\frac{3}{2}\)

\(\frac{1}{y\left(y+1\right)}+\frac{y}{2}+\frac{y+1}{4}\ge\sqrt[3]{\frac{1}{y\left(y+1\right)}.\frac{y}{2}.\frac{y+1}{4}}=3.\sqrt{\frac{1}{4}}=\frac{3}{2}\)

\(\frac{1}{z\left(z+1\right)}+\frac{z}{2}+\frac{z+1}{4}\ge\sqrt[3]{\frac{1}{z\left(z+1\right)}.\frac{z}{2}.\frac{z+1}{4}}=3.\sqrt{\frac{1}{4}}=\frac{3}{2}\)

\(\Rightarrow\frac{1}{x\left(x+1\right)}+\frac{x}{2}+\frac{x+1}{4}\)\(+\frac{1}{y\left(y+1\right)}+\frac{y}{2}+\frac{y+1}{4}\)

\(+\frac{1}{z\left(z+1\right)}+\frac{z}{2}+\frac{z+1}{4}\ge\frac{3}{2}.3=\frac{9}{2}\)

\(\Leftrightarrow\frac{1}{x^2+x}+\frac{1}{y^2+y}+\frac{1}{z^2+z}+\frac{x+y+z}{2}+\frac{x+y+z+3}{4}\ge\frac{9}{2}\)

\(\Leftrightarrow\frac{1}{x^2+x}+\frac{1}{y^2+y}+\frac{1}{z^2+z}+\frac{3}{2}+\frac{3}{2}\ge\frac{9}{2}\)

\(\Leftrightarrow\frac{1}{x^2+x}+\frac{1}{y^2+y}+\frac{1}{z^2+z}\ge\frac{3}{2}\left(đpcm\right)\)